shaynfernandez
Thanks Received: 5
Elle Woods
Elle Woods
 
Posts: 91
Joined: July 14th, 2011
 
 
 

Q19 - Light is registered in the

by shaynfernandez Fri Jul 29, 2011 3:14 pm

This is definitely one of the hardest LR questions i have encountered on my PT's I am having a hard time understanding why B is the correct answer. I do not see where the premises favors warm temperature being more error-prone. There does not even seem to be a mention of the warmth, or lack there of, of the temperature.
 
timmydoeslsat
Thanks Received: 887
Atticus Finch
Atticus Finch
 
Posts: 1136
Joined: June 20th, 2011
 
This post thanked 1 time.
 
trophy
Most Thanked
trophy
First Responder
 

Re: Q19 - Light is registered in the

by timmydoeslsat Fri Jul 29, 2011 3:38 pm

shaynfernandez Wrote:This is definitely one of the hardest LR questions i have encountered on my PT's I am having a hard time understanding why B is the correct answer. I do not see where the premises favors warm temperature being more error-prone. There does not even seem to be a mention of the warmth, or lack there of, of the temperature.


There is not an explicit mention of warmth, but there is a mention of temperature of the retina.

This is not an argument. It is a set of statements.

When photons hit pigment R and change R's shape ---> light is registered in retina

The next statement talks about how R's shape can be changed without the presence of those light photons. R's shape can be changed because of normal molecular motion.

Think of this as a simple force of nature that produces the same kind of result or effect as a light photon would. These statements also tell us that when the R molecule changes shape, it introduces error into the visual system.

The last statement of this stimulus makes this particular force of nature more clear. Without this last statement, all we know is that R's molecular shape can change just because of a natural process called normal molecular motion.

I wonder if there are any factors that can EVEN ALTER THE NUMBER OF OCCURRENCES THAT MOTION?

There is! "The amount of this molecular motion is directly proportional to the temperature of the retina."

So, for it to be directly proportional, we know that the more molecular motion there is, the more temperature we have, which in "everyday talk" means warmer.

We know that if we are in a hot surrounding, the visual system will be more error prone than in a cold one. This is because a hot surrounding has warmer temperatures than the cold surroundings. This means that the amount of molecular motion is higher in the hot surroundings than in the cold surroundings. This means that more error is introduced because error is introduced when the R molecule changes shape.
 
shaynfernandez
Thanks Received: 5
Elle Woods
Elle Woods
 
Posts: 91
Joined: July 14th, 2011
 
 
 

Re: Q19 - Light is registered in the retina when photons hit mol

by shaynfernandez Thu Aug 18, 2011 11:17 pm

These types of questions are passages are often frustrating because the LSAT teaches you to not use "outside knowledge" of course i know that motion would indicate a rise in temperature, but am i supposed to use that information to relate to this LSAT passage, or should i simply base it on the exact knowledge that is given in the stimulus.
 
timmydoeslsat
Thanks Received: 887
Atticus Finch
Atticus Finch
 
Posts: 1136
Joined: June 20th, 2011
 
 
trophy
Most Thanked
trophy
First Responder
 

Re: Q19 - Light is registered in the retina when photons hit mol

by timmydoeslsat Fri Aug 19, 2011 11:33 am

shaynfernandez Wrote:These types of questions are passages are often frustrating because the LSAT teaches you to not use "outside knowledge" of course i know that motion would indicate a rise in temperature, but am i supposed to use that information to relate to this LSAT passage, or should i simply base it on the exact knowledge that is given in the stimulus.


In this question particularly, there is no outside knowledge needed.

Notice that choice B doesn't even require us to know or infer anything about eye retina temperatures and the possibility of those retinas being a different temperature in warmer/colder environments.

B states "visual systems of animals THAT MATCHES that of their surroundings. So we now know that since motion is directly proportional to temperature of the retina, we know that more of temperature is more of motion, and more of motion is more introduction of error into visual system.


In general, the LSAT is only going to use the most basic outside knowledge, if any is to be brought, for its questions. And I would argue that this "outside knowledge" is really the most helpful in terms of knowing that "phrase 1 in the stimulus" equates to "phrase 2 in an answer choice."

For example, if the stimulus mentioned gravity, but an answer choice referred to it as a natural phenomenon or something similar, that is the sort of outside knowledge needed in my opinion.
User avatar
 
bbirdwell
Thanks Received: 864
Atticus Finch
Atticus Finch
 
Posts: 803
Joined: April 16th, 2009
 
This post thanked 1 time.
 
 

Re: Q19 - Light is registered in the retina when photons hit mol

by bbirdwell Sat Aug 20, 2011 12:08 pm

So, for it to be directly proportional, we know that the more molecular motion there is, the more temperature we have, which in "everyday talk" means warmer.


Yes, timmydoeslsat! Knowing the explicit meaning of "directly proportional" as opposed to "inversely proportional" is a big help in understanding that high temps = high motion.

Even this is not needed to correctly answer the question, though.

Here are the facts:

photons hit rhodopsin, change its shape ---> light registered

motion ----> rhodopsin change shape = errors

temperature --> motion
_________________
Now, let's put the facts together and make any 1+2=3 kinds of conclusions before going to the choices, if possible.

Just follow the chain backwards and connect the dots! Temp leads to motion, leads to changing shape, leads to errors. So:

change in temperature of retina can cause errors

With this in mind, now look at the choices.

A) light changes temperature? Seems like I get hotter in the sun, but the text doesn't say anything about this. Therefore it's unsupported and we eliminate.

B) More errors in hot than cold? Hmm. Does body temp = retina temp? This choice probably seems too specific at first. At least it's about temperature and errors, which is the key "inference" we made up front. Even if we don't love it, we should leave it for now for that reason alone, without exerting too much energy scrutinizing it on our first pass through the choices.

C) speed of rhodopsin reactivity? Not discussed.

D) sensitivity of rhodopsin? Not discussed.

E) ONLY naturally occurring molecules? Unsupported.

Whoa. Must be B.

Hope that helps! Great discussion, guys...
I host free online workshop/Q&A sessions called Zen and the Art of LSAT. You can find upcoming dates here: http://www.manhattanlsat.com/zen-and-the-art.cfm
User avatar
 
LSAT-Chang
Thanks Received: 38
Atticus Finch
Atticus Finch
 
Posts: 479
Joined: June 03rd, 2011
 
 
trophy
Most Thankful
trophy
First Responder
 

Re: Q19 - Light is registered in the retina when photons hit mol

by LSAT-Chang Sat Sep 03, 2011 1:33 pm

I always wondered if a sentence that had "is directly proportional" could be translated into a conditional. I noticed you did above, so if we have "A is directly proportional to B" is it B-->A? How does this logic work?
 
zee.brad
Thanks Received: 3
Forum Guests
 
Posts: 28
Joined: February 02nd, 2012
 
 
 

Re: Q19 - Light is registered in the retina when photons hit mol

by zee.brad Sun Feb 19, 2012 11:43 am

changsoyeon Wrote:I always wondered if a sentence that had "is directly proportional" could be translated into a conditional. I noticed you did above, so if we have "A is directly proportional to B" is it B-->A? How does this logic work?


In math, "A is directly proportional to B" can be expressed as "A ∝ B", means when B increases, A increases accordingly, I am not sure whether or not its a conditional relationship but rather a causal.
User avatar
 
bbirdwell
Thanks Received: 864
Atticus Finch
Atticus Finch
 
Posts: 803
Joined: April 16th, 2009
 
 
 

Re: Q19 - Light is registered in the retina when photons hit mol

by bbirdwell Wed Feb 22, 2012 2:16 am

interesting question. Maybe something like this:

If A is directly proportional to B:
It would have to both ways.
A increase <--> B increase

And also:
A decrease <--> B decrease

it kind of works in terms of notation, though technically it's a correlation, not necessarily a causation.
I host free online workshop/Q&A sessions called Zen and the Art of LSAT. You can find upcoming dates here: http://www.manhattanlsat.com/zen-and-the-art.cfm
 
griffin.811
Thanks Received: 43
Atticus Finch
Atticus Finch
 
Posts: 127
Joined: September 09th, 2012
 
 
 

Re: Q19 - Light is registered in the

by griffin.811 Thu Jan 10, 2013 11:36 pm

I understand the idea of motion being directly proportional, but what's frustrating to me is that the passage never says that the MORE molecular motion we have, the MORE prone to errors. Rather it just says normal molecular motion is a contributing factor to errors.

I guess my question is why isn't it possible that cooler temperatures, causing a reduction in movement, also cause error?

The way I see it, our motion is in the middle of the line
-----x-----

if temp goes up, motion up, and error (so we are further up the spectrum).
------------x--

but, similarly, temp down, motion down, error
--x--------------

so (B) should say something like "...are more error prone in hot surroundings than the temp they are USED TO."

why is this way of thinking incorrect?
User avatar
 
bbirdwell
Thanks Received: 864
Atticus Finch
Atticus Finch
 
Posts: 803
Joined: April 16th, 2009
 
This post thanked 1 time.
 
 

Re: Q19 - Light is registered in the

by bbirdwell Tue Jan 15, 2013 12:47 pm

so (B) should say something like "...are more error prone in hot surroundings than the temp they are USED TO."


I can't see why it would need to say "used to." As you pointed out, when the retina is hotter, there's more error.

(B) says their bodies match surroundings. So, when surroundings are hotter, bodies (ie retinas) are hotter, and errors would be higher.

I guess my question is why isn't it possible that cooler temperatures, causing a reduction in movement, also cause error?


It might be possible, but the facts we have do not state this. And our job is to use the facts we have. The facts we have actually support the opposite:

Motion is proportional to temp. So when temp is low, motion is low, errors are low. When temp is high, motion is high, errors are high.
I host free online workshop/Q&A sessions called Zen and the Art of LSAT. You can find upcoming dates here: http://www.manhattanlsat.com/zen-and-the-art.cfm
 
griffin.811
Thanks Received: 43
Atticus Finch
Atticus Finch
 
Posts: 127
Joined: September 09th, 2012
 
 
 

Re: Q19 - Light is registered in the

by griffin.811 Sun Jan 20, 2013 11:39 am

bbirdwell Wrote:
so (B) should say something like "...are more error prone in hot surroundings than the temp they are USED TO."


I can't see why it would need to say "used to." As you pointed out, when the retina is hotter, there's more error.

(B) says their bodies match surroundings. So, when surroundings are hotter, bodies (ie retinas) are hotter, and errors would be higher.

I guess my question is why isn't it possible that cooler temperatures, causing a reduction in movement, also cause error?


It might be possible, but the facts we have do not state this. And our job is to use the facts we have. The facts we have actually support the opposite:

Motion is proportional to temp. So when temp is low, motion is low, errors are low. When temp is high, motion is high, errors are high.


Thanks again bbirdwell, this makes perfect sense after reading your explanation.

I think my error stemmed from misunderstanding the word PROPORTIONAL.
 
patrice.antoine
Thanks Received: 35
Atticus Finch
Atticus Finch
 
Posts: 111
Joined: November 02nd, 2010
 
 
 

Re: Q19 - Light is registered in the

by patrice.antoine Thu Feb 21, 2013 1:07 pm

While I get motion is proportional to temperature who is to say a rise in temperature introduces error into visual system? Unless LSAT wants us to use outside information in assuming that light produces heat and such effects are similar to when "molecules change shape due to normal molecular motion" ??

The whole "hot" and "cold" introductions in the correct answer choice threw me off.
 
tzyc
Thanks Received: 0
Atticus Finch
Atticus Finch
 
Posts: 323
Joined: May 27th, 2012
 
 
trophy
Most Thankful
 

Re: Q19 - Light is registered in the

by tzyc Sat Mar 30, 2013 10:44 pm

Still have a problem with (C)...
I think in cold surroundings there still can be temperature changes.
Such as...from very cold to a little cold...
How do we know from the stimulus that more error happens in hot surrounding than cold surrounding?
I think we cannot say either is more error prone...
Or...is it because there are few animals which body temperature matches cold surroundings? (few has cold temprature body?)
Then is this an ouside knowledge because the stimulus does not say there are fewer cold body temperature animals...
This question is really confusing :(
 
srusing
Thanks Received: 0
Forum Guests
 
Posts: 2
Joined: October 30th, 2013
 
 
 

Re: Q19 - Light is registered in the

by srusing Wed Oct 30, 2013 3:37 pm

I HATE this question, please help!

In order to make the proper assumptions and get to the right answer for this question, one has to hold that the biochemical properties of the eye are reduced to the simplicity of "molecular motion" within a pot of water boiling and freezing. Less molecular motion results from less heat and vice versa, more energy results in more motion, fine, this is acceptable.

The unreasonable assumption is drawn when we must connect the change in molecular motion with an assumption that the change in error is greater in a hot, rather than cold environment. This may be true, but why should I assume it is? To come back to my water example, my water pipes may freeze or boil, due to "molecular motion" but which will cause more damage to my piping system? Whether the change is proportional or not, there is simply no statement that qualifies how the "change" because of "molecular motion", the addition or subtraction of heat, will affect the error rate of the rhodopsin? All that is known is that they "change" in accordance with the assumptions of "normal molecular motion." The water in my pipes also "changes", due to "normal molecular motion," and it can even be "directly proportional" but how does any of this information tell me boiling pipes results in a worse off system than freezing pipes?

To sum up my issue, the question ask's you to assume knowledge of a biochemical process that claims having ice in your eye is less problematic for vision than having steam in your eye. Where am I wrong?
User avatar
 
tommywallach
Thanks Received: 468
Atticus Finch
Atticus Finch
 
Posts: 1041
Joined: August 11th, 2009
 
 
 

Re: Q19 - Light is registered in the

by tommywallach Wed Oct 30, 2013 11:53 pm

Hey Sr,

I don't understand your point at all.

Molecules move more when it's hot. The passage says the more heat there is, the more problems. So we conclude (B), there are more problems in hot surroundings if the heat actually makes the eye hot (which is what (B) says). I have no idea where your metaphor came from!

-t
Tommy Wallach
Manhattan LSAT Instructor
twallach@manhattanprep.com
Image
 
srusing
Thanks Received: 0
Forum Guests
 
Posts: 2
Joined: October 30th, 2013
 
 
 

Re: Q19 - Light is registered in the

by srusing Thu Oct 31, 2013 12:44 am

t

The passage says rhodopsin "sometimes changes" because of "normal molecular motion" and that the amount is directly proportional to the temp of the retina. From this we are supposed to deduce that in a hotter environment, the molecular motion will cause more error. Can you please explain why this deduction is logically valid?
User avatar
 
tommywallach
Thanks Received: 468
Atticus Finch
Atticus Finch
 
Posts: 1041
Joined: August 11th, 2009
 
 
 

Re: Q19 - Light is registered in the

by tommywallach Mon Nov 04, 2013 3:18 pm

Hey Sr,

No, you're still misunderstanding it. The passage already told us DEFINITIVELY that heat causes error. Heat is, by definition, the increased motion of molecules. So if the molecules are moving more quickly, there will be more errors.

-t

Dictionary to the rescue: Heat: a form of energy arising from the random motion of the molecules of bodies...
Tommy Wallach
Manhattan LSAT Instructor
twallach@manhattanprep.com
Image
 
olivialynne
Thanks Received: 0
Vinny Gambini
Vinny Gambini
 
Posts: 1
Joined: November 12th, 2015
 
 
 

Re: Q19 - Light is registered in the

by olivialynne Thu Nov 12, 2015 12:35 pm

I just did this test as a PT and....

My question is when did we start talking about "animals" in the stem? There is no mention of animals until the answer choices, so that threw me off...Help?

Thanks!
 
ShennelS204
Thanks Received: 0
Vinny Gambini
Vinny Gambini
 
Posts: 1
Joined: June 06th, 2017
 
 
 

Re: Q19 - Light is registered in the

by ShennelS204 Tue Jun 06, 2017 3:56 pm

tommywallach Wrote:Hey Sr,

No, you're still misunderstanding it. The passage already told us DEFINITIVELY that heat causes error. Heat is, by definition, the increased motion of molecules. So if the molecules are moving more quickly, there will be more errors.

-t

Dictionary to the rescue: Heat: a form of energy arising from the random motion of the molecules of bodies...


Thank you so much! This question was giving me so much trouble. Your explanation made me completely understand my error
 
vstoever
Thanks Received: 3
Vinny Gambini
Vinny Gambini
 
Posts: 22
Joined: March 02nd, 2017
 
 
 

Re: Q19 - Light is registered in the

by vstoever Sat Jul 22, 2017 5:26 pm

I think what is very helpful here is to have a clear understanding of the meaning of directly proportional and inversely proportional and surprisingly I have found that these two terms come up several times, for example, also on this same section #1 (you need to know meaning of varies inversely on answer choice).

If you understand that directly proportional means if A up then B up, or temp goes up then molecular motion goes up then you will understand the answer.

The opposite would be inversely proportional, if one goes up the other goes down, which is not stated.

So we know that as it gets hotter, (temp up), molecular motion goes up which it states can cause errors. :)

I have not studied math since I was in high school so I had to look up difference between these two, but it does seem LSAT expects you to know these two terms, hope this helps.